LSAT and Law School Admissions Forum

Get expert LSAT preparation and law school admissions advice from PowerScore Test Preparation.

 LSAT2018
  • Posts: 242
  • Joined: Jan 10, 2018
|
#46288
I understand why A is parallel to the stimulus but I would just like to clarify whether there is valid or invalid reasoning here. In stating that A is directly proportional to B, would they be interchangeable, like working either way? So would it be acceptable to say B increased, A would increase.


Thanks in advance!
 Adam Tyson
PowerScore Staff
  • PowerScore Staff
  • Posts: 5153
  • Joined: Apr 14, 2011
|
#63251
Yes, LSAT2018, if two things are directly proportional then you can say that as either one of them increased, so did the other, and as either of them decreased, so did the other. If you accept the claim of direct proportion, then you can use changes in either one to prove changes in the other. That concept comes up now and then on the test, so it's good to be familiar with it, and also with indirect or inverse proportions, where they move in opposite directions.
User avatar
 conorrjohnston
  • Posts: 9
  • Joined: Jan 16, 2024
|
#104967
Hi!!

I got this one right, but I wanted to see why D was out. My original reasoning was that D didn't have the *insert numeric quantity* increase portion of the stimulus, and that it redirects the benefits from the subject (students) to another subject (the university seeing new fringe benefits) whereas the stimulus doesn't have this subject redirection. I think D could be valid if it said something to the effect of "Since the number of students at the Athletic Institute is increasing by 20 percent, it follows that students fees will increase". Is this a correct rationale?

Thanks!
 Robert Carroll
PowerScore Staff
  • PowerScore Staff
  • Posts: 1787
  • Joined: Dec 06, 2013
|
#105038
conorrjohnston,

The change in subject in answer choice (D) is from the number of classes in which a particular student enrolls to the number of students enrolled. It's not shifting from student benefits to institute benefits. But it does have a change of subject which makes this answer incorrect.

Robert Carroll

Get the most out of your LSAT Prep Plus subscription.

Analyze and track your performance with our Testing and Analytics Package.